solve the equation x 1.)132.)13/33.) 104.) none of these choices

Solve The Equation X 1.)132.)13/33.) 104.) None Of These Choices

Answers

Answer 1

Answer:

2. 13/3

Step-by-step explanation:

Answer 2

x will be equal to 13/3.

Given,

5^(2x - 1) = 5^(5x - 14)

We can see that base is the same for both the exponents on each side of the equation.

Now, on using the Logarithm on both sides with base 5, we can see that the base on both sides of the equation cancels out with the log (base 5) function.

And new equation becomes:

(2x - 1) = (5x - 14)

This derives us to another conclusion that if the base of an exponent is equal then,

the powers must be equal too.

(2x - 1) = (5x - 14)

=> 5x - 2x = -1 + 14

=> 3x = 13

which gives us,

=> x = 13/3.

Therefore x = 13/3.

Learn more about Exponential Equations at

https://brainly.com/question/250321


Related Questions

Help in writing an equation. I believe that it is supposed to be a linear equation

Answers

Since the information required us that the equation has to start in zero we can think of functions like the root of x but also we have to add a value of 1/3. In other words one equation with those characteristics is

[tex]y=\sqrt{x}+\frac{1}{3}[/tex]

Graph the line y = -4 on the graph below.

Answers

we have the equation

y=-4

This is a horizontal line (parallel to the x-axis) that passes through the point (0,-4)

see the graph below to better understand the problem

2. The area A of a rectangle is represented by the formula A = Lw, where Lis the length and wis the width. The length of the rectangle is 5. Write anequation that makes it easy to find the width of the rectangle if we knowthe area and the length.

Answers

[tex]w=\frac{A}{5}[/tex]

1) Considering that the Area of a rectangle is given as:

[tex]A=lw[/tex]

2) We can then write the following equation plugging into that the length= 5.

Say the area is "A", then we can find the width this way:

[tex]\begin{gathered} A=5ww \\ 5w=A \\ \frac{5w}{5}=\frac{A}{5} \\ w=\frac{A}{5} \end{gathered}[/tex]

Note that we rewrote that to solve it for w (width).

All we need is to plug into the A the quantity of the area of this rectangle

Thus, the answer is w=A/5

The width of a rectangle is [tex] \frac{3}{4} [/tex] its length. The perimeter of the rectangle is 420 ft. What is the length, in feet, of the rectangle?

Answers

The width of a rectangle is 3/4 its length.

[tex]w=\frac{3}{4}l[/tex]

The perimeter of the rectangle is 420 ft.

Recall that the perimeter of a rectangle is given by

[tex]P=2(w+l)[/tex]

Let us substitute the value of the given perimeter and the width

[tex]\begin{gathered} P=2(w+l) \\ 420=2(\frac{3}{4}l+l) \end{gathered}[/tex]

Now simplify and solve for length

[tex]\begin{gathered} 420=2(\frac{3}{4}l+l) \\ 420=\frac{3}{2}l+2l \\ 420=3.5l \\ l=\frac{420}{3.5} \\ l=120\: ft \end{gathered}[/tex]

Therefore, the length of the rectangle is 120 feet.

Slope =
y-intercept = (0,

Answers

Answer:

y intercept= (0,-3)

slope= 2/1 or simplified 2

Step-by-step explanation:

There is a 50% chance of rain here and a 10% chance of rain on Mars. Therefore, there is a 45% chance that it will rain in neither place.

Answers

The statement that " There is a 45% chance that it will rain in neither place" is true.

In the question ;

it is given that

Probability of raining here = 50% = 0.5

Probability of raining on mars = 10% = 0.1

So, the probability of not raining here = 1-0.5 = 0.5

and probability of not raining on mars = 1-0.1 = 0.9

Hence the probability of rain in neither place = (probability of not raining here)×(probability of not raining on mars) .

Substituting the values , we get

probability of rain in neither place = 0.5×0.9

= 0.45

= 45%

Therefore , the statement " There is 45% chance that it will rain in neither place" is true.

The given question is incomplete , the complete question is

There is a 50% chance of rain here and a 10% chance of rain on Mars. Therefore, there is a 45% chance that it will rain in neither place.

Is the statement True or False ?

Learn more about Probability here

https://brainly.com/question/27942798

#SPJ1

Jake and Joshua have new jobs selling gift cards at a local convenience store at the cash register, but their pay is different. Jake earns a foundational wage of $6 per hour, as well as $8 for each gift card sold. Joshua gets $4 for each gift card sold and earns a foundational wage of $6 per hour. If they each sell a certain number of gift cards in one hour, they will end up earning the same amount of pay. How many gift cards would that make up to?Write a system of equations, graph them, and type the solution.

Answers

Let x be the number of cards Jake and Joshua sell within one hour. Therefore, their earnings are given by the following expressions,

[tex]\begin{gathered} Ja=6+8x \\ Jo=6+4x \end{gathered}[/tex]

Then, set Ja=Jo (both earn the same amount),

[tex]\begin{gathered} Ja=Jo \\ \Rightarrow6+8x=6+4x \end{gathered}[/tex]

Solving for x,

[tex]\begin{gathered} \Rightarrow8x=4x \\ \Rightarrow4x=0 \\ \Rightarrow x=0 \end{gathered}[/tex]

Then, they will earn the same within one hour only if both sell zero cards within the hour.

Graphing the system of equations,

As one can see, the intersection point is (0,6), which stands for 0 cards and $6

1 litre=1000cm³. About how many test tubes, each holding 24cm³ of water, can be filled from a
1 litre flask?

Answers

Answer: 125/3 or about 41.667

Note that you can't have 2/3 of a test tube, so the expected answer may be 42 test tubes.

Step-by-step explanation:

Write a simple algebra equation using the word problem

24x = 1000

x represents the number of test-tubes, each of which hold 24cm^3 of water.  

divide both sides by 24

x = 125/3 or about 41.667

Look at this graph: 100 90 80 60 50 20 10 10 20 30 50 60 70 80 90 100 What is the slope? Simplify your answer and write it as a proper fraction, improper fraction or integer. Submit Not feeling yet These con heo

Answers

To find the slope of the line we have to use this equation:

[tex]m=\frac{y_2-y_1}{x_2-x_1}[/tex]

Now we have to replace two coordiantes in the line, so I was able to see the coordinates: (0,40) and (20,50), sothe equation become:

[tex]m=\frac{50-40}{20-0}[/tex]

and we simplify so:

[tex]m=\frac{10}{20}=\frac{1}{2}[/tex]

So the slope is 1/2

I inserted a picture of the question Check all that apply

Answers

Recall that the line equation is of the form

[tex]y=mx+c\ldots\ldots\text{.}(1)[/tex]

The points lie in the line are (2,5) and (-2,-5).

Setting x=2 and y=5 in the equa

If the expression 1/ square root of x was placed in form x^a, then which of the following would be the value of a?

Answers

4) -1/2

1) Rewriting the expression:

[tex]\frac{1}{\sqrt[]{x}}[/tex]

2) As a power we can write this way, considering that we can rewrite any radical as a power and that when we have a radical on the denominator we can rewrite it as a negative rational exponent. So we can write it out:

[tex]\frac{1}{\sqrt[]{x}}=\frac{1}{x^{\frac{1}{2}}}=x^{-\frac{1}{2}}[/tex]

3) Hence, the answer is 4) -1/2

3450 turns to degrees and 3450 turns to radians.

Answers

We will have the following:

*First: We know that 1 turn will be equal to 360°. So:

[tex]3450\cdot360=1242000[/tex]

So, 3450 turns equal to 1 242 000 degrees.

*Second: We have that the expression to convert degrees to radians is:

[tex]d\cdot\frac{\pi}{180}=r[/tex]

Here d represents degrees and r radians. So, we replace the number of degrees and solve for radians:

[tex](1242000)\cdot\frac{\pi}{180}=6900\pi[/tex]

So, 3450 turns are 6900pi radians.

I need help with this question please help me asap?

Answers

Answer

Explanation

• Range (R): the amount between the upper and lower limit.

Use a system of equations to solve the following problem.The sum of three integers is380. The sum of the first and second integers exceeds the third by74. The third integer is62 less than the first. Find the three integers.

Answers

the three integers are 215, 12 and 153

Explanation:

Let the three integers = x, y, and z

x + y + z = 380 ....equation 1

The sum of the first and second integers exceeds the third by 74:

x + y - 74 = z

x + y - z = 74 ....equation 2

The third integer is 62 less than the first:

x - 62 = z ...equation 3

subtract equation 2 from 1:

x -x + y - y + z - (-z) = 380 - 74

0 + 0 + z+ z = 306

2z = 306

z = 306/2

z = 153

Insert the value of z in equation 3:

x - 62 = 153

x = 153 + 62

x = 215

Insert the value of x and z in equation 1:

215 + y + 153 = 380

368 + y = 380

y = 380 - 368

y = 12

Hence, the three integers are 215, 12 and 153

HelppppppFunction f is a(n)functionThe graph is a reflection in thewith a verticaland atranslationunits:The domain of f isThe domain of the parent function is;The range of f isThe range of the parent function is

Answers

Answer:

In order of appearance of  boxes

quadraticx-axisstretch3 (units)upall real numbersall real numbersy ≤ 3y ≥ 0

Step-by-step explanation:

The given function f(x) = -2x² + 3 belongs to the quadratic family of equations. A quadratic equation has a degree of 2. The degree is the highest power of the x variable in the function f(x)

The parent f(x) = x²

Going step by step:

2x²  ==> graph x² is vertically stretched by 2. For any value of x in x², the new y value is twice that the old value. For example, in the original parent function x², for x = 2, y = 4. In the transformed function 2x², for x = 2, y = 2 x 4 = 8 so it has been stretched vertically. It becomes skinnier compared to the original

-2x²  => graph is reflected over the x-axis. It is the mirror image of the original graph when viewed from the x-axis perspective

-2x² + 3 ==> graph is shifted vertically up by  3 units

Domain is the set of all x-input values for which the function is defined. For both x² and -2x² + 3 there are no restrictions on the values of x. So the domain for both is the set of all real numbers usually indicated by
-∞ < x < ∞

The range is the set of all possible y values for a function y = f(x) for x values in domain.

The range of f(x) = x² is x≥ 0 since x² can never be negative

Range of -2x² + 3 is  x ≤ 3 : Range of -2x² is y ≤ 0 since y cannot be negative and therefore range of -2x² + 3 is y ≤ 3

What is the slope of a line perpendicular to the line whose equation is15x + 12y = -108. Fully reduce your answer.Answer:Submit Answer

Answers

GIven:

The equation of a line is 15x+12y=-108.

The objective is to find the slope of the perpencidular line.

It is known that the equation of straight line is,

[tex]y=mx+c[/tex]

Here, m represents the slope of the equation and c represents the y intercept of the equation.

Let's find the slope of the given equation by rearranging the eqation.

[tex]\begin{gathered} 15x+12y=-108 \\ 12y=-108-15x \\ y=-\frac{15x}{12}-\frac{108}{12} \\ y=-\frac{5}{4}x-9 \end{gathered}[/tex]

By comparing the obtained equation with equation of striaght line, the value of slope is,

[tex]m_1=-\frac{5}{4}[/tex]

THe relationship between slopes of a perpendicular lines is,

[tex]\begin{gathered} m_1\cdot m_2=-1 \\ -\frac{5}{4}\cdot m_2=-1 \\ m_2=-1\cdot(-\frac{4}{5}) \\ m_2=\frac{4}{5}^{} \end{gathered}[/tex]

Hence, the value of slope of perpendicular line to the given line is 4/5.

the stock market lost 231 points on Tuesday then walks 128 more points on Wednesday find a change of points over the two days

Answers

the change of the points is:

[tex]-231-128=-359[/tex]

so in the 2 days the stock market lost 359 points

According to Debt.org the average household has $7,281 in credit card debt. Estimate how much interest the average household accumulates over the course of 1 year. We are going to assume the APR is 16.99%.

Answers

In order to estimate the interest the average househould accumulates in 1 year, you use the following formula:

A = Prt

where P is the initial credit card debt ($7,281), r is the interest rate per period (16.99%) and t is the number of time periods. In this case the value of r is given by the APR, then, there is one period of 1 year.

To use the formula it is necessary to express 16.99% as 0.1699. Thus, you have:

I = 7,281 x 0.1699 x 1

I = 1,237.04

Hence, the interest accumulated is of $1,234.04

Carlos is saving money to buy a new Nintendo Switch game. He has $25. After he receives his allowance (n), he will have $45. Which of the following equations models this situation?

Answers

ANSWER

25 + n = 45

EXPLANATION

We have that Carlos already has $25.

His allowance is n. After receiving it, he now has $45.

This means that if we add the amount he had and his allowance, we will have $45.

Therefore:

25 + n = 45

This equation models the situation accurately.

I think I’m off to a good start but I’m still confused

Answers

Given

The radius is given 3.5 ft and height is given 14 ft.

Explanation

To find the surface area of cylinder,

Use the formula.

[tex]S=2\pi rh+2\pi r^2[/tex]

Substitute the values.

[tex]\begin{gathered} S=2\pi r(h+r) \\ S=2\times3.14\times3.5(14+3.5) \\ S=384.65ft^2 \end{gathered}[/tex]

The volume of cylinder is determined as

[tex]V=\pi r^2h[/tex]

Substitute the values

[tex]\begin{gathered} V=3.14\times3.5^2\times14 \\ V=538.51ft^3 \end{gathered}[/tex]

Answer

The surface area of cylinder is 384.65 sq.ft.

The volume of cylinder is 538.51 cubic feet.

[tex]f(x) = \sqrt{ {x }^{2} - 121} [/tex]what's the inverse of this equation

Answers

EXPLANATION:

The first thing to do in the equation is to interchange the variables in order to do the inverse function and thus solve the equation correctly.

So the equation is as follows:

[tex]\begin{gathered} x=\sqrt[]{x^2-121} \\ Now\text{ we exchange }the\text{ variables x and y;} \\ x=\sqrt[]{y^2-121} \\ y=\sqrt[]{x^2+121};\text{ y}=-\sqrt[]{x^2+121} \\ \sqrt[]{x^2+121\text{ ; }-\sqrt[]{x^2+121}} \\ \text{the answer is : }\sqrt[]{x^2+121\text{ },\text{ }}\text{ }-\sqrt[]{x^2+121^{}} \end{gathered}[/tex]

Which of the following could be the points that Jamur plots?

Answers

To solve this problem, we need to calculate the midpoint for the two points in each option and check if it corresponds to the given midpoint (-3,4).

Calculating the midpoint for the two points of option A.

We have the points:

[tex](-1,7)and(2,3)[/tex]

We label the coordinates as follows:

[tex]\begin{gathered} x_1=-1 \\ y_1=7 \\ x_2=2 \\ y_2=3 \end{gathered}[/tex]

And use the midpoint formula:

[tex](\frac{x_1+x_2}{2},\frac{y_1+y_2}{2})[/tex]

Substituting our values:

[tex](\frac{-1_{}+2_{}}{2},\frac{7_{}+3_{}}{2})[/tex]

Solving the operations:

[tex](\frac{1_{}}{2},\frac{10_{}}{2})=(\frac{1_{}}{2},5)[/tex]

Since the midpoint is not the one given by the problem, this option is not correct.

Calculating the midpoint for the two points of option B.

We have the points:

[tex](-2,6)and(-4,2)[/tex]

We follow the same procedure, label the coordinates:

[tex]\begin{gathered} x_1=-2 \\ y_1=6 \\ x_2=-4 \\ y_2=2 \end{gathered}[/tex]

And use the midpoint formula:

[tex]\begin{gathered} (\frac{x_1+x_2}{2},\frac{y_1+y_2}{2}) \\ \text{Substituting our values} \\ (\frac{-2-4_{}}{2},\frac{6+2_{}}{2}) \\ \text{Solving the operations:} \\ (\frac{-6}{2},\frac{8}{2}) \\ (-3,4) \end{gathered}[/tex]

The midpoint for the two points in option B is (-3,4) which is the midpoint given by the problem.

Answer: B (-2,6) and (-4,2)

the bearing from S to R is 160° what is the bearing of S from R

Answers

.

The bearing of S from R is given as;

[tex]90+90+90+70=340\degree[/tex]

As an incoming college freshman, Tina received a 10-year $15,100 Federal Direct
Unsubsidized Loan with an interest rate of 4.29%. She knows that she can begin making
loan payments 6 months after graduation but interest will accrue from the moment the
funds are credited to his account. How much interest will accrue while she is still in
school and over the 6-month grace period for this freshman year loan?
O $2,813.28
O $2,915.06
O $3,001.32
O $3,102.38

Answers

The interest that will accrue while the college freshman is still in school and over the 6-month grace period for this freshman year loan is, approximately, B. $2,915.06.

How is the interest determined?

The interest for federal college loans is based on the simple interest formula instead of compounding.

By compounding, we mean that interest is computed on the principal and accumulated interest.

Federal Direct Unsubsidized Loan = $15,100

Number of years for college = 4 years

The number of years before repayment = 4.5 years (4 years + 6 months)

Simple interest for 4.5 years = $2,915.06 ($15,100 x 4.29% x 4.5 years)

On the other hand, one can compute the compounded interest using an online finance calculator.

Compounded Interest:

N (# of periods) = 4.5 years

I/Y (Interest per year) = 4.29%

PV (Present Value) = $15,100

PMT (periodic payment) during the 4.5 years = $0

Results:

FV = $18,241.85

Total Interest = $3,141.85

Thus, the interest is Option B.

Learn more about simple interests at https://brainly.com/question/2294792

#SPJ1

Find the area of this trapezoid. Be sure to include the correct un4 cm6 cm4 cm15 cm

Answers

So,

Here we have the following trapezoid:

Remember that the area of a trapezoid can be found if we apply the following formula:

[tex]A=\frac{1}{2}(\text{base}1+\text{base}2)\cdot\text{height}[/tex]

Where bases 1 and 2 are the greater and smaller bases respectively.

So, if we replace:

[tex]\begin{gathered} A=\frac{1}{2}(15+4)\cdot4 \\ A=\frac{1}{2}(19)\cdot4 \\ A=9.5\cdot4 \\ A=38 \end{gathered}[/tex]

So the area is 38cm^2.

A genetic experiment with
peas resulted in one sample of offspring that consisted of 447 green peas and 169 yellow peas.
a. Construct a 90% confidence interval to estimate of the percentage of yellow peas.
b. Based on the confidence interval, do the results of the experiment appear to contradict the expectation that 25% of the offspring peas would be yellow?
a. Construct a 90% confidence interval. Express the percentages in decimal form.
L s p< (Round to three decimal places as needed.)
b. Based on the confidence interval, do the results of the experiment appear to contradict the expectation that 25% of the offspring peas would be yellow?
O
No, the confidence interval includes 0.25, so the true percentage could easily equal 25%
L
O Yes, the confidence interval does not include 0.25, SO the true percentage could not equal 25%

Answers

Using the z-distribution, it is found that:

a. The 90% confidence interval to estimate of the percentage of yellow peas is: (34.04%, 41.58%).

b. The correct option is: Yes, the confidence interval does not include 0.25, so the true percentage could not equal 25%.

What is a confidence interval of proportions?

The bounds of a confidence interval of proportions is given according to the equation presented as follows:

[tex]\pi \pm z\sqrt{\frac{\pi(1-\pi)}{n}}[/tex]

In which the parameters are described as follows:

[tex]\pi[/tex] is the sample proportion.z is the critical value of the distribution.n is the sample size, from which the estimate was built

The confidence level is of 90%, hence the critical value is z = 1.645, using a z-distribution calculator.

The values of the sample size and of the estimate are given as follows:

[tex]n = 447, \pi = \frac{169}{447} = 0.3781[/tex]

Hence the lower bound of the interval is:

[tex]\pi - z\sqrt{\frac{\pi(1-\pi)}{n}} = 0.3781 - 1.645\sqrt{\frac{0.3781(0.6219)}{447}} = 0.3404[/tex]

The upper bound is:

[tex]\pi + z\sqrt{\frac{\pi(1-\pi)}{n}} = 0.3781 + 1.645\sqrt{\frac{0.3781(0.6219)}{447}} = 0.4158[/tex]

As a percentage, the interval is given as follows: (34.04%, 41.58%).

The confidence interval does not contain 0.25, hence the true percentage would not be equal to 25%, contradicting the expectation.

A similar problem, also involving the z-distribution, is given at https://brainly.com/question/25890103

#SPJ1

question 5 only. determine the missing side length QP. the triangles are not drawn to scale.

Answers

This is a simple question.

First, we can see both triangles are proportional, it means it has the same relation between its sides even if one is in a large scale and the other on a a small scale.

Now we can identify which side corresponds to which side. Once side AC is the longest one for triangle ABC it means its equivalent for triangle PQR is the side RP, so the equivalent for side AB is side QP. Once we know that we can write the following relation and calculate:

What does slope mean?

Answers

Slope is a measure of its steepness

Mathematically,

Slope = Rise / Run

Rise = y2 - y1

Run = x2 - x1

Slope = y2 - y1 / x2 - x1

Answer:

Suppose a linear equation describes something (say, population growth). The slope is the rate (say, of growth) and the y-intercept gives the starting value.

Step-by-step explanation:

Iq scores were gathered for group of college students at a local university. What is the level of measurement of dataNominal, ordinal, interval, ratio

Answers

Nominal data refers to non numerical data, for example categories, colors, etc...

Ordinal data refers to numerical data with a natural order, it comprehends real numbers.

Intervals comprehends data with equal distance between the values and no meaningful zero

Ratios comprehends data with equal distance between the values and a meaningul zero value.

With this in mind, the IQ scores of the college students represent numerical data, with a natural order, and the distance between the values is not equal, so you can classify the data as "ordinal"

Solve the system using the elimination method:2x - y + z = -26x + 3y - 4z = 8-3x + 2y + 3z = -6

Answers

multiply 2x - y + z = - 2 for 3

[tex]6x-3y+3z=-6[/tex]

then sunstract the equation 1 and 2

[tex]\begin{gathered} 6x+3y-4z=8 \\ 6x-3y+3z=-6 \\ 6y-7z=14 \end{gathered}[/tex]

multiply -3x+2y+3z=-6 for 2

[tex]-6x+4y+6z=-12[/tex]

adding

[tex]\begin{gathered} -6x+4y+6z=-12 \\ \underline{6x-3y+3z=-6} \\ y+9z=-18 \end{gathered}[/tex]

multiply y+9z=-18 for 6

[tex]6y+54z=-108[/tex]

Subtracting

[tex]\begin{gathered} 6y+54z=-108 \\ \underline{6y-7z=14} \\ 61z=-122 \end{gathered}[/tex]

then solve

[tex]\begin{gathered} 61z=-122 \\ \frac{61z}{61}=\frac{-122}{61} \\ z=-2 \end{gathered}[/tex][tex]\begin{gathered} 6y-7\mleft(-2\mright)=14 \\ 6y+14=14 \\ 6y+14-14=14-14 \\ 6y=0 \\ y=0 \end{gathered}[/tex][tex]\begin{gathered} 6x-3\cdot\: 0+3\mleft(-2\mright)=-6 \\ 6x-6=-6 \\ 6x-6+6=-6+6 \\ 6x=0 \\ x=0 \end{gathered}[/tex]

answer is: x = 0, y = 0 and z = - 2

Other Questions
when hasbro comes up with an idea for a new toy, it needs to ensure that the children who play with the toy will not choke on any of its parts and that the toy in general will not be a hazard to the children who will be using it. in what stage of the new-product development process will these safety checks occur? Triangle VWX is formed by connecting the midpoints of the side of triangle STU.The lengths of the sides of triangle VWX are shown. What is the length of SU?Figures not necessarily drawn to scale. Find the coordinates of the center, vertices, covertices, foci, length of transverse and conjugate axis and the equation of the asymptotes. Then graph the hyperbola. Line g is dilated by a scale factor of 1/2 from the origin to create line g'. Where are points E' and F' located after dilation, and how are lines g and g' related? 4 3 g 1 E -5 -2 -1 0 -1 -2 Type the appropriate form of the verb in the blank Mr. Rodriguez is preparing photos for an international client. The client has requested a photo that is 20 cm by 15 cm. Mr. Rodriguez knows that the formula c = 2.54n can be used to convert n inches to c centimeters. Which formula can he use to convert centimeters to inches? what are the problems that the people on the main street are facing in the story called the monsters are due on main street Lily likes to collect records. Last year she had 12 records in her collection. Now she has 15 records. What is the percent increase ofher collection?The percent increase of her collection is cyclical unemployment is the group of answer choices portion of unemployment that is due to the normal working of the labor market. unemployment that results when people become discouraged about their chances of finding a job so they stop looking for work. portion of unemployment that is due to changes in the structure of the economy that result in a significant loss of jobs in certain industries. unemployment that occurs during recessions and depressions. 3. Craft and Structure: How does the reporter structure the article to present through Laurent the central idea that students are being punished unfairly? The perimeter of a rectangle is 36 cm and the length is twice the width. What are the dimensions of this rectangle? Whats the length and width? which of the following is the most fundamental issue that economics addresses? responses choice of appropriate technology choice of appropriate technology reduction of unemployment reduction of unemployment reduction of budget deficit reduction of budget deficit promotion of privatization promotion of privatization use of scarce resources I need help with this question... the correct answer choice Graph A) -f(x) B) f(x+2) -4Then find the domain and range of each For which equations is 8 a solution? Select the four correct answers. x + 6 = 2 x + 2 = 10 x minus 4 = 4 x minus 2 = 10 2 x = 4 3 x = 24 StartFraction x Over 2 EndFraction = 16 StartFraction x Over 8 EndFraction = 1 . you want to design a breathing machine that will bring 500 ml of air into the lungs. you begin with 100 ml of air at 2 atm of pressure. how would you change the pressure in your machine to accomplish your goal? asapcompare and contrast chemical and physical equilibrium Perimeter and area polynomials In the ________ activity of scm, organizations coordinate the receipt of customer orders, develop a network of warehouses, select carriers to transport their products to their customers, and create an invoicing system to receive payments. Find the volume of this cylinder. Use 3 for a.14 ftV = 7r2h=9 ftVV ~ [?]ft3